0
$\begingroup$

enter image description here

Attempt

Let $x_1$ be the number of hours to produce product 1 during assembly and $x_2$ be the number of hours to produce product 1 during finishing. Notice that $\frac{1}{2} x_1 + x_2 $ is the number of product 1 units produced. Similarly, call $x_3$ and $x_4$ be the number of hours spent during assembly and finishing to produce product two. We see thtat the number of units of product 2 is $x_3 + 2 x_4$. We want to maximize profit. So, our objective function is

$$ z = 25 (0.5 x_1 + x_2) + 15 (x_3+2x_4) $$

subject to

$$ x_1 + x_3 \leq 120 $$

$$ x_2 + x_4 \leq 85 $$

$$ x_1,x_2,x_3,x_4 \geq 0 $$

To find a worth of an hour of assembly and an hour of finishing, do we need to find the dual solution? Since the dual varaibles (y_1,y_2) that corresponds to each constraint one measures hours of assembly and $y_2$ measures hours of finishing. But, what is the units of the dual objective? Am I on the right track?

$\endgroup$

1 Answer 1

2
$\begingroup$

I would use two variables, $z_1, z_2$ to be the number of chairs to be made

$$\max 25 z_1 + 15z_2$$

subject to

$$2z_1 + z_2 \le 120$$

$$z_1 + \frac12z_2 \le 85$$

$$z \ge 0$$

Notice that the finishing constraint is never active.

$\endgroup$
6
  • $\begingroup$ Thanks for the reply. So the answer to this problem would then be just the dual variables correct ? $\endgroup$ Commented Nov 8, 2018 at 8:20
  • $\begingroup$ hmm... unlikely. It can't be that we are not paying someone at all. Salary negotiation in my interpretation is a "game", depending on who knows what and the market situation. We know that a single chair is earning us $\$15$ and that takes $1$ hour of assembly and $\frac12$ hour of finishing. $\endgroup$ Commented Nov 8, 2018 at 8:25
  • $\begingroup$ So how can we find then the worth of an hour assembly and finishing workers time ? $\endgroup$ Commented Nov 8, 2018 at 8:27
  • $\begingroup$ i do not know. I think most likely I misinterpreted the question. I keep viewing the question as a game rather than a single player problem. Is the question asking for shadow price? $\endgroup$ Commented Nov 8, 2018 at 8:30
  • $\begingroup$ The problem is on duality section of my notes, so I guess its related to shadow prices. $\endgroup$ Commented Nov 8, 2018 at 8:35

You must log in to answer this question.

Start asking to get answers

Find the answer to your question by asking.

Ask question

Explore related questions

See similar questions with these tags.